With which one of the following statements would the author be most likely to agree?

Ryan on October 24, 2019

Why is C correct? Why is D incorrect?

Thanks

Reply
Create a free account to read and take part in forum discussions.

Already have an account? log in

Irina on October 24, 2019

@Ryan-Mahabir,

(C) is supported by lines 45-51 "..in reality whether a patent ..would be enforced ..is questionable. First, patent litigation is an expensive endeavor..." The author explicitly lists the cost of litigation as one of the reasons patents would not be enforced in a case of basic non commercial research.

(D) is incorrect because even though it is true that biotechnology researchers rely on research funding that is in part conditional on patent ability of their results (lines 4-7), the author never argues that they rely on such funding too heavily, arguably there are other sources of funding as well for basic research.